Вы находитесь на странице: 1из 33

1

CHAPTER
REAL NUMBER

 DIVISIBILITY
A non-zero integer ‘a’ is said to divide an integer ‘b’ if their exists an integer c such that b = ac.

The integer ‘a’ is called divisior, integer ‘b’ is known as the dividend and integer ‘c’ is known as
the quotient.

For example, 3 divides 36 because there is an integer 12 such that 36 = 3 × 12. However, 3 does not
divide 35 because there do not exist an integer c such that 35 = 3 × c, In other words, 35 = 3 × c is
not true for any integer c.

If a non-zero integer ‘a’ divides an integer b, then we write a|b. This is read as “a divides b”. When
a|b, we say that ‘b is divisible by a’ or ‘a is a factor of b’ or ‘b is a multiple of a’ or ‘a is a divisor of b’.

We observe that
(i) –4 | 20, because there exists an integer –5 such that 20 = –4 × (–5).

(ii) 4 | –20, because there exists an integer –5 such that –20 = 4 × (–5).
(iii) –4 | –20, because there exists an integer 5 such that –20 = –4 × 5.

Following are some properties of divisibility:

(i) ± 1 divides every non-zero integer.


i.e., ± 1 | a for every non-zero integer a.

(ii) 0 is divisible every non-zero integer a.


i.e., a | 0 for every non-zero integer a.

(iii) 0 does not divide any integer.

(iv) If a is a non-zero integer and b is any integer, then a|b  a|–b, –a|b and –a|–b

(v) If a and b are non-zero integers, then a | b and b | a  a = ± b

(vi) If a is a non-zero integer and b, c are any two integers, then

a|b  c 
 
a | b and a | c  3  a|bc  for any integer x.
 a|bx 
 

(vii) If a and c are non-zero integers and b, d are any two integers, then

(a) a | b and c | d  ac | bd
(b) ac | bc  a | b

IIT ASHRAM UG–1 & 2, Concorde Complex, Above OBC Bank. R.C. Dutt Road., Alkapuri Baroda.
Page # 1
Real Numbers (Class X) Mathematics

 EUCLID’S DIVISION LEMMA


Consider the division of one positive integer by another, say 58 by 9. The division can be carried
out as follows:

9 ) 58 (6
54
4

While carrying out this division, we had to think of about the largest multiple of 9 which does not
exceed 58 so that after subtraction the remainder 4 is less than the divisor 9. The result of this
division is that we get two integers, namely, 6 which is called the quotient and 4 which is called
the remainder. We can write the result in the following form:

58 = 9 × 6 + 4, 0  4 < 9

Let us now apply the same procedure to other pairs of positive integers to see whether such a
representation is always possible or not.

Pair of integers Representation

 7 goes into 25 thrice and leaves 


25, 7  25 = 7 × 3 + 4, 0  4 < 7  remainder 4 
 

 3 goes into 20 six times leaves 


20, 3  20 = 3 × 6 + 2, 0  2 < 3  remainder 4 
 

15 is larger than 7. So, this 


7, 15  7 = 15 × 0 + 7, 0  7 < 15  relat ion is always possible 
 

 5 goes into 35 seven times 


35, 5  35 = 5 × 7 + 0, 0  0 < 15  and leaves no remainder 
 

It is evident from the above discussion that the above representation also holds for other pairs of
integers. We also observe that for each pair of positive integers a and b, we can find unique integers
q and r satisfying the relation a = bq + r, 0  r < b. In fact, this holds for every pair of positive
integers as proved in the following lemma.

 EUCLID’S DIVISION LEMMA


Let a and b be any two positive integers. Then, there exist unique integers q and r such that

a = bq + r, 0  r < b

If b | a, then r = 0. Otherwise, r satisfies the stronger inequality 0< r < b.


PROOF - Consider the following arithmetic progression .......,

a – 3b, a – 2b, a – b, a, a + b, a + 2b, a + 3b, .......


Clearly, it is an arithmetic progression with common difference ‘b’ and it extends indefinitely in
both directions.

Let r be the smallest non-negative then of this arithmetic progression. Then, there exists a non-
negative integer q such that a – bq = r  a = bq + r

IIT ASHRAM UG–1 & 2, Concorde Complex, Above OBC Bank. R.C. Dutt Road., Alkapuri Baroda.
Page # 2
Real Numbers (Class X) Mathematics

As, r is the smallest non-negative integer satisfying the above result. Therefore, 0  r < b

Thus, we have a = bq + r, where 0  r < b

We shall now prove the uniqueness of q and r.

Uniqueness To prove the uniqueness of q and r, let us assume that there is another pair q1 and r1
of non-negative integers satisfying the same relation i.e., a = bq1 + r1, 0  r1 < b
We shall now prove that r1 = r and q1 = q

We have,

a = bq + r and a = bq1 + r1

 bq + r = bq1 + r1  r1 – r = bq – bq1  r1 – r = b(q – q1)

 b | r1 – r

 r1 – r = 0 [  0  r < b and 0  r1 < b Þ 0  r1 – r < b]

 r1 = r
Now, r1 = r

 –r1 = – r [Multiplying both side by (–1)]

 a – r1 = a – r [Adding a on both sides]

 a  bq  r and a  bq1  r1 
 bq1 = bq  
 bq  a  r and bq1  a  r1 

 q1 = q

Hence, the representation a = bq + r, 0  r < b is unique.

Ex. Show that every positive even integer is of the form 2q, and that every positive odd integer is of the
form 2q +1, where q is some integer.

Sol. Let a be any positive integer and b = 2. Then, by Euclid’s division Lemma there exist integers q and
r such that
a = 2q + r, where 0  r < 2

Now, 0  r < 2  0  r  1  r = 0 or, r = 1 [  r is an integer]

 a = 2q or, a = 2q + 1

If a = 2q, then a is an even integer.

We know that an integer can be either even or odd. Therefore, any odd integer is of the form
2q + 1.

Ex. Show that n2 – 1 is divisible by 8, if n is an odd positive integer.

Sol. We know that any odd positive integer is of the form 4q + 1 or, 4q + 3 for some integer q.

So, we have the following cases:


In this case, we have

n2 – 1 = (4q + 1)2 – 1 = 16q2 + 8q + 1 – 1 = 16q2 + 8q = 8q (2q + 1)

IIT ASHRAM UG–1 & 2, Concorde Complex, Above OBC Bank. R.C. Dutt Road., Alkapuri Baroda.
Page # 3
Real Numbers (Class X) Mathematics

 n2 – 1 is divisible by 8 [  8q (2q + 1) is divisible by 8]

Case II When n = 4q + 3

In this case, we have


n2 – 1 = (4q + 3)2 – 1 = 16q2 + 24q + 9 – 1 = 16q2 + 24q + 8

 n2 – 1 = 8 (2q2 + 3q + 1) = 8 (2q + 1) (q + 1)
 n2 – 1 is divisible by 8 [  8(2q + 1) (q + 1) is divisible by 8]

Hence, n2 – 1 is divisible by 8.

DPP -1
(QUESTIONS ON EUCLID’S DIVISION LEMMA)1QQ1
1. Show that any positive integer is of the form 3q or, 3q + 1 or, 3q + 2 for some integer q.

2. Prove that n2 – n is divisible by 2 for every positive integer n.

3. Show that the square of any positive integer is of the form 3m or, 3m + 1 for some integer m.

4. Use Euclid’s division Lemma to show that the cube of any positive integer is either of the form
9m, 9m + 1 or, 9m + 8 for some integer m.

5. Show that one and only one out of n, n + 2 or, n + 4 is divisible by 3, where n is any positive integer.

6. Show that any positive odd integer is of the form 6q+1 or, 6q+3 or, 6q + 5, where q is some integer.
7. Prove that if a positive integer is of the form 6q + 5, then it is of the form 3q + 2 for some integer q,
but not conversely.

8. Prove that one of every three consecutive positive integers is divisible by 3.

9. If a and b are two odd positive integers such that a > b, then prove that one of the two number

ab a b
and is odd and the other is even
2 2
10. Prove that the product of two consecutive positive integers is divisible by 2.

11. Show that the square of any positive integer is of the form 4q or 4q + 1 for some integer q.

12. Show that every positive even integer is of the form 2q, and that every positive odd integer is of the
form 2q + 1, where q is some integer .

13. Show that any positive odd integer is of the form 4q + 1 or 4q + 3, where q is some integer.
14. Show that the square of an odd integer is of the form 4q + 1 for some integer q.

15. Show that the cube of a positive integer is of the form 6q + r, q is an integer and r = 0, 1, 2, 3, 4, 5 is
also of the form 6m + r.
16. For any positive integer n, prove that n3 – n is divisible by 6.

17. Show that one and only one out of n, n + 4, n + 8, n + 12 and n + 16 is divisibleby 5, where n is any
positive integer.
18. Prove that if x and y are both odd positive integers, then x2 + y2 is even but not divisible by 4.

IIT ASHRAM UG–1 & 2, Concorde Complex, Above OBC Bank. R.C. Dutt Road., Alkapuri Baroda.
Page # 4
Real Numbers (Class X) Mathematics

 EUCLID’S DIVISION ALGORITHM


An algorithm means a series of well defined steps which provide a procedure of calculation repeated
successively on the results of earlier steps till the desired result is obtained. Euclid’s division
algorithm is also an algorithm to compute the highest common factor (HCF) of two given positive
integers. If an integer c divides each one of several integers x 1, x2,....xn, then it is called a common
divisor of these integers. For example, 7 is a common divisor of 42 and 63 as it divides both the
integers.

Theorem 1If a and b are positive integers such that a = bq + r, then every common divisor of a and b is
a common divisor of b and r, and vice-versa.
Proof : Let c be a common divisor of a and b. Then,

c | a  a = cq1 for some integer q1

c | b  b = cq2 for some integer q2

Now, a = bq + r

 r = a – bq  r = cq1 – cq2q  r = c(q1 – q2q)  c|r


 c | r and c | b [  c | b (given)]  c is a common divisor of b and r.

Hence, a common divisor of a and b is a common divisor of b and r.

Conversely, Let d be a common divisor of b and r. Then,

d | b  b = r1 d for some integer r1

d | r  r = r2 d for some integer r2


We will now show that d is a common divisor of a and b.

We have, a = bq + r

 a = r1dq + r2d  a = (r1q + r2)d

 d|a  d | a and d | b [  d | b (given)]

 d is a common divisor of a and b.


Let us now discuss an application of this theorem and Euclid’s division lemma. Consider integers
117 and 45.

Let a = 117 and b = 45

By Euclid’s division lemma, we have  45 ) 117 (2


 
117 = 45 × 2 + 27 ...(i)  90 
 27 
or, a = bq1 + r1, where q1 = 2 and r1 = 27 

By using the above theorem, we observe that the common divisors of a = 117 and b =45 are also the
common divisors of b = 45 and r1 = 27 and vice-versa.

Applying Euclid’s division lemma on divisor b = 45 and remainder r2 = 27, we get

45 = 27 × 1 + 18 ...(ii)
 27 ) 45 (1
 
 27 
 18 

IIT ASHRAM UG–1 & 2, Concorde Complex, Above OBC Bank. R.C. Dutt Road., Alkapuri Baroda.
Page # 5
Real Numbers (Class X) Mathematics

or, b = q2r1 + r2, where q2 = 1 and r2 = 18

Using the above theorem, we find that the common divisors of r1 = 27 and r2 = 18 are the common
divisors of b = 45 and r1 = 27 and vice-versa. But, common divisors of b = 45 and r1 = 27 are the
common divisors of a = 117 and b = 45 and vice-versa. Therefore, common divisors of r1 = 27 and
r2 = 18 are the common divisors of a = 117 and b = 45 and vice-versa.

Applying Euclid’s division lemma on r1 = 27 and r2 = 18, we get

 18) 27 (1
 
27 = 18 × 1 + 9 ...(iii)  18 
 9 

or, r1 = q3r2 + r3, where q3 = 1 and r3 = 9


Again by using the above theorem, we find that common divisors of r2 = 18 and r3 = 9 are the
common divisors of a = 117 and b = 45 and vice-versa.
Using Euclid’s division lemma on r2 = 18 and r3 = 9, we get

18 = 9 × 2 + 0 ...(iv)

Therefore, r3 = 9 is a divisor of r2 = 18 and r3 = 9. Also, it is the greatest common divisor (or HCF) of
r2 and r3. Hence, r3 = 9 is the greatest common divisor (or HCF) of a = 117 and b = 45. We also
observe that r3 = 9 is the last non-zero remainder in the above process of repeated application of
Euclid’s division lemma on the divisor and the remainder in the next step.

The set of equation (i) to (iv) is called Euclid’s division algorithm for 117 and 45. The last divisor, or
the last but one non-zero remained which is 9 is the HCF (or GCD) of 117 and 45. The above
process of finding HCF can also be carried out by successive divisons as follows:

45) 117 (2
90
27 ) 45 (1
27 45 117
1 2
27 90
18 ) 27 (1
18 27
18 OR, 2 1
18 18
9 ) 18 (2 00 9
18
00

Theorem 2 : (Fundamental Theorem of Arithmetic) Every composite number can be expressed (factorised)
as a product of primes, and this factorization is unique except for the order in which the
prime factors occur. While writing a positive integer as the product of primes, if we decide to
write the prime factors in ascending order and we combine the same primes, then the integer
is expressed as the product of power of primes and the representation is unique. So, we can
say that every composite number can be expressed as the products of powers distinct primes
in ascending or descending order in a unique way.

IIT ASHRAM UG–1 & 2, Concorde Complex, Above OBC Bank. R.C. Dutt Road., Alkapuri Baroda.
Page # 6
Real Numbers (Class X) Mathematics

Following theorem is a direct consequence of the Fundamental Theorem of Arithmetic.

 SOME APPLICATIONS OF THE FUNDAMENTAL THEOREM OF ARITHMETIC


 Finding hcf and lcm of positive integers

In order to find the HCF and LCM of two or more positive integers, we may use the following
algorithm.

 Algorithm

STEP - I : Factorize each of the given positive integers and express them as a product of powers of
primes in ascending order of magnitudes of primes.

STEP - II : To find the HCF, identify common prime factors and find the smallest (least) exponent of
these common factors. Now raise these common prime factors to their smallest exponents and
multiply them to get the HCF.

To find the LCM, list all prime factors (once only) occuring in the prime factorization of the given
positive integers.

For each of these factors, find the greatest exponent and raise each prime factor to the greatest
exponent and multiply them to get the LCM.

 Determining the nature of the decimal expansions of rational number


Theorem 3. Let x be a rational number whose decimal expansion terminates. Then, x can expressed in
p
m
the form
q , where p and q are co-primes, and the prime factorisation of q is of the form 2 ×
5n, where m, n are non-negative integers.

Let us now see whether the converse of this theorem is also true or not. That is, if we have a
p
m n
rational number of the form
q , and the prime factorisation of q is of the form 2 × 5 , where
p
m, n are non-negative integers, then does
q have a terminating decimal?

a
Let be a rational number in the lowest form such that the prime factorisation of b is of the
b
form 2m× 5n, where m,n are non-negative integers.

We have the following cases:


Case I : When m = n :

a a a a
In this case, we have  m n
 m m

b 2 5 2 5 (10)m
Case II: When m > n :

In this case, we have


m = n + p, where p is a positive integer.

a a a  5p a  5p a  5p c
 m n
 m n p
 m m
 m
 m , where c = a × 5p
b 2 5 2 5 2 5 (2  5) 10
Case III: When m < n :

IIT ASHRAM UG–1 & 2, Concorde Complex, Above OBC Bank. R.C. Dutt Road., Alkapuri Baroda.
Page # 7
Real Numbers (Class X) Mathematics

In this case, we have

n = m + p, where p is positive integer.

a a a  2p a  2p a  5p c
 m n
 mp n
 n n
  , where c = a × 2p
b 2 5 2 5 2 5 (2  5)n 10n

Thus, a rational number whose denominator is of the form 2m × 5n, where m, n are non-negative
c
integers, can be converted to an equivalent rational number of the form , where d is a power of
d
10.

 For example,

7 7 7  53 7  125 875 189 189 23  189 8  189 1512


(i)  3  3   (ii)  3  3  
8 2 2  5 3 (2  5)3 103 125 5 2  53 (2  5)3 10 3

49 49 49  2 98 98
(iii)  2 3
 3 3
 3
 3
500 2  5 2 5 (2  5) 10

p
Theorem 4 : Let x  q be a rational number, such that the prime factorisation of q is of the form 2m × 5n,
where m, n are non-negative integers. Then, x has a decimal expansion which terminates.

Let us now consider rational numbers whose decimal expansions are non-terminating and
repeating. For example,

5 17 1
(i)  1.66666........ (ii)  2.8333.......... (iii)  0.142857142857.........
3 6 7

We observe that the prime factorisation of the denominators of these rational numbers are
not of the form 2m × 5n, where m, n are non-negative integers.

So,we arrive at the following conclusion.


p
Theorem 5. Let x  be a rational number, such that the prime factorisation of q is not of the form 2m ×
q
5n, where m,n are non-negative integers. Then, x has a decimal expansion which is non-
terminating repeating.

Let us now discuss some examples to determine the nature of the decimal expansions of
rational numbers by using the above theorems.

Ex. Use Euclid’s division algorithm to find the HCF of 4052 and 12576.

Sol Given integers are 4052 and 12576 such that 12576 > 4052. Applying Euclid’s division lemma to
12576 and 4052, we get

 4052) 12576 (3 
 
12576 = 4052 × 3 + 420 ...(i)  12156 
 420 
 

Since the remainder 420  0. So, we apply the division lemma to 4052 and 420, to get

IIT ASHRAM UG–1 & 2, Concorde Complex, Above OBC Bank. R.C. Dutt Road., Alkapuri Baroda.
Page # 8
Real Numbers (Class X) Mathematics

 420 ) 4052 (9 
 
 3780 
4052 = 420 × 9 + 272 ...(ii)
 272 

We consider the new divisor 420 and the new remainder 272 and apply divison lemma to get

 272) 420 (1


 
 272 
420 = 272 × 1 + 148 ...(iii)
 148 

Let us now consider the new divisor 272 and the new remainder 148 and apply division lemma to
get

 148) 272 (1


 
 148 
272 = 148 × 1 + 124 ...(iv)
 124 

We consider now the new divisor 148 and the new remainder 124 and apply division lemma to get

 124) 148 (1


 
 124 
148 = 124 × 1 + 24 ...(v)
 24 

We consider now the new divisor 124 and the new remainder 24 and apply division lemma to get

 24) 124 (5 
 
 120 
124 = 24 × 5 + 4 ...(vi)
 4 
 

We consider the new divisor 24 and the new remainder 4 and apply division lemma to get

 4) 24 (6 
 
 24 
24 = 4 × 6 + 0 ...(vii)
 0 

We observe that the remainder at this stage is zero. Therefore, the divisor at this stage i.e. 4 (or
the remainder at the earlier stage) is the HCF of 4052 and 12576.

Ex. Find the largest number that will divide 398, 436 and 542 leaving remainders 7, 11 and 15
respectively.

Sol Clearly, the required number is the HCF of the numbers

398 – 7 = 391, 436 – 11 = 425, and, 542 – 15 = 527.

First we find the HCF of 391 and 425 by Euclid’s algorithm as given below.

IIT ASHRAM UG–1 & 2, Concorde Complex, Above OBC Bank. R.C. Dutt Road., Alkapuri Baroda.
Page # 9
Real Numbers (Class X) Mathematics

391 425
11 1
374 391
17 34
2
(HCF) 34
(Remainder)

Clearly, H.C.F. of 391 and 425 is 17.

Let us now the HCF of 17 and the third number 527 by Euclid’s algorithm :

17 527
3
(HCF) 51
17
1
17
0
(Remainder)

The HCF of 17 and 527 is 17. Hence, HCF of 391, 4250 and 527 is 17.

Hence, the required number is 17.


Ex. Determine the prime factorization of each of the following numbers :
(i) 13915 (ii) 556920
Sol. (i) Using the prime factorization tree, we have

13915 = 5 × 11 × 11 × 23 = 5 × 112 × 23

(ii) Using the prime factorization tree, we have

556920 = 2 × 2 × 2 × 3 × 3 × 5 × 7 × 13 × 17 = 23 × 32 × 5 × 7 × 13 × 17

IIT ASHRAM UG–1 & 2, Concorde Complex, Above OBC Bank. R.C. Dutt Road., Alkapuri Baroda.
Page # 10
Real Numbers (Class X) Mathematics

Ex. Find the HCF and LCM of 144,180 and 192 by prime factorisation method.

Sol. Using the factor tree for the prime factorisation of 144, 180 and 192,

we have

144 = 24 × 32,180 = 22 × 32 × 5and 192 = 26 × 3

To find the HCF, we list the common prime factors and their smallest exponents in 144,180 and
192 as follows:

Common prime factors Least exponents

2 2

3 1

HCF = 22 × 31 =12
To find the LCM, we list all prime factors of 144, 180, 192 and their greatest exponents as follows:

Prime factors of 144,180 and 192 Greatest exponents

2 6
3 2

5 1

LCM = 26 × 32 × 51 =64 × 9 × 5 = 2880

Ex. In a seminar, the number of participants in Hindi, English and Mathematics are 60,84 and 108
respectively. Find the minimum number of rooms required if in each room the same number of
participants are to be seated and all of them being in the same subject.

Sol. The Number of room will be minimum if each room accmodates maximum number of participates.
Since in each room the same number of participants ar e to be seated and all of them must be of
the same subject. Therefore, the number of participants in each room must be the HCF of 60, 84
and 108. The prime factorisations of 60, 84 and 108 are as under:

60 = 22 × 3 × 5, 84 = 22 × 3 × 7 and 108 = 22 × 32

 HCF of 60, 84 and 108 is 22 × 3 = 12


Therefore, in each room 12 participants can be seated.
Total number of participants 60  84  108 252
 Number of rooms required = =   21
12 12 12

Ex. There is a circular path around a sports field. Priya takes 18 minutes to drive one round of the
field, while Ravish takes 12 minutes for the same. Suppose they both start at the same point and
at the same time, and go in the same direction. After how many minutes will they meet again at
the starting point?
Sol. Required number of minutes is the LCM of 18 and 12.

We have, 18 = 2 × 32 and 12 = 22 × 3

 LCM of 18 and 12 is 22 × 32 = 36.

Hence, Ravish and Priya will meet again at the starting point after 36 minutes.
IIT ASHRAM UG–1 & 2, Concorde Complex, Above OBC Bank. R.C. Dutt Road., Alkapuri Baroda.
Page # 11
Real Numbers (Class X) Mathematics

DPP - 2
(QUESTIONS OF EUCLID’S DIVISION ALGORITHM)
1. Use Euclid’s division algorithm to find the HCF of 210 and 55.

2. Use Euclid’s division algorithm to find the HCF of

(i) 135 and 225 (ii) 176 and 38220 (iii) 567 and 255.
3. Find the HCF of 595 and 107, using Euclid’s algorithm.

4. Find HCF of 180, 252 and 324 using Euclid’s Division algorithm.

5. Use Euclid’s algorithm to find the HCF of 4052 and 12576.

6. Find the HCF 84 and 105, using Euclid’s algorithm.

7. Find the HCF of 861 and 1353, using Euclid’s algorithm.


8. Find the HCF of 616 and 1300, using Euclid’s algorithm.

9. Use Euclid’s division algorithm to find the HCF of

(i) 13281 and 15844 (ii) 1128 and 1464

(iii) 4059 and 2190 (iv) 10524 and 12752


(v) 10025 and 14035

10. Use Euclid’s division algorithm to find the HCF of 441, 567, 693.

(QUESTIONS ON LINEAR COMBINATION (ax + by)

11. Find the HCF of 81 and 237 and express it as a linear combination of 81 and 237.

12. If d is the HCF of 56 and 72, find x,y satisfying d = 56x + 72y. Also show that x and y are not unique.
13. If the HCF of 408 and 1032 if expressible is the form 1032m – 408 × 5. Find m

14. If the HCF of 65% and 963 is expressible in the form 657x + 963 × (–15), find x

ANSWERS
1. 5 2. (i) 45, (ii) 4, (iii) 63 3. 1 4. 36 5. 4

6. 21 7. 123 8. 4 9. (i) 233 (ii) 24 (iii) 3 (iv) 4 (v) 2005.

10. 63 11. 3 12. x = 4, y = – 3 13. m = 2 14. x = 22

Ex. Prove that 3 is an irrational number.


Sol. Let us assume on the contrary that 3 is a rational number. Then, there exist positive integers
a
a and b such that 3 , where a and b are co-prime i.e. their HCF is 1.
b
a a2
Now, 3  3  3b2 = a2
b b2
 3 | a2 [  3 | 3b2]

 3|a [By Theorem 2 on page 1.25] ...(i)

 a = 3c for some integer c

IIT ASHRAM UG–1 & 2, Concorde Complex, Above OBC Bank. R.C. Dutt Road., Alkapuri Baroda.
Page # 12
Real Numbers (Class X) Mathematics

 a2 = 9c2  3b2 = 9c2 [  a2 = 3b2]

 b2 = 3c2  3 | b2 [  3 | 3c2]

 3|b [By Theorem 2 on page 1.25] ...(ii)

From (i) and (ii), we observe that a and b have at least 3 as a common factor. But this contradicts
the fact that a and b are co-prime. This means that our assumption is not correct.

Hence, 3 is an irrational number.

Ex. Prove that 3 + 2 5 is rational. Then there exist co-prime positive integers a and b such that

Sol. Let us assume on the contrary that 3 + 2 5 is rational. Then there exist co-prime positive integers
a and b such that

a a a  3b
32 5   2 5  3  5
b b 2b

 a  3b 
 5 is rational  a, b are integers  2b
is a rational 
 
This contradicts the fact that 5 is irrational. So, our supposition is incorrect.

Hence, 3  2 5 is an irrational number.

Ex. What can you say about the prime factorisations of the denominators of the following rationals:

(i) 34.12345 (ii) 34.5678


Sol. (i) Since 34.12345 has terminating decimal expansion. So, its denominator is of the form 2m ×
5n, where m, n are non-negative integers.

(ii) Since 34.5678 has non-terminating repeating decimal expansion. So, its denominator has
factors other than 2 or 5.

DPP - 3
(QUESTIONS ON FUNDAMENTAL THEOREM OF ARITHMETIC AND PRIME FACTORISATION)

1. Express each of the following positive integers as the product of its prime factors:
(i) 140 (ii) 156 (iii) 234

2. Find the HCF and LCM of 90 and 144 by the prime factorisation method.
3. Find the HCF of 96 and 404 by prime factorisation method. Hence, find their LCM.

4. Prove that there is no natural number for which 4n ends with the digit zero.

5. Check whether 6n can end with the digit 0 for any natural number n.
6. Find the LCM and HCF of 15, 18, 45 by prime factorisation method.

7. Show that 5n can’t end with the digit 2 for any natural number n.
8. Show that 9n can’t end with the digit 2 for any integer n.

9. Find the prime factorization of the following numbers :

(i) 10000 (ii) 2160 (iii) 396 (iv) 4725 (v) 1188

IIT ASHRAM UG–1 & 2, Concorde Complex, Above OBC Bank. R.C. Dutt Road., Alkapuri Baroda.
Page # 13
Real Numbers (Class X) Mathematics

10. Find the missing numbers in the following factorisation :

11. Find the prime factors of the following numbers :

(i) 1300 (ii) 13645 (iii) 3456

12. Split 4536 and 18511 into their prime factors and hence find their LCM and HCF.
13. Find the LCM of the following using prime factorization method. :

(i) 72, 90, 120 (ii) 24, 63, 70 (iii) 455, 117, 338
(iv) 225, 240, 208 (v) 2184, 2730, 3360

14. Show that 12n cannot end with the digit 0 or 5 for any natural number n.

15. Explain why 7 × 11 × 13 + 13 and 7 × 6 × 5 × 4 × 3 × 2 × 1 + 5 are composite numbers.


16. Explain why 3 × 5 × 7 + 7 is a composite number.

17. Find the LCM and HCF of 6 and 20 by the prime factorisation method.

18. Find the HCF of 12576 and 4052 by using the prime factorisation method.

19. Find the HCF and LCM of 6, 72 and 120 using the prime factorisation method.

ANSWERS
1. (i) 22 × 5 × 7, (ii) 22 × 3 × 13, (iii) 2 × 32 × 13 2. 18,720
3. 4,9696 5. No 6. LCM = 90, HCF = 3
4 4 4 3 2 2 3 2
9. (i) 2 × 5 ; (ii) 2 × 3 × 5 ; (iii) 2 × 3 × 11 (iv) 3 × 5 × 7 (v) 22 × 33 × 11
10. (a) 4800 (b) 2400 (c) 1200 (d) 600 (e) 300 (f) 150
(g) 75 (h) 25
11. (i) 2 × 5 × 13 (ii) 5×2729 (iii) 27 × 33
2 2
12. 149688, 567
13. (i) 360 (ii) 2520 (iii) 106470 (iv) 46800 (v) 43680 17. 60, 2 18. 4
19. 6, 360

DPP - 4
(QUESTIONS ON L.C.M/H.C.F)
1. The HCF of two numbers is 145 and their LCM is 2175. If one number is 725, find the other.
2. Find the smallest number which leaves remainders 8 and 12 when divided by 28 and 32 respectively.

3. What is the smallest number that, when divided by 35, 56 and 91 leaves remainders of 7 in each
case ?

IIT ASHRAM UG–1 & 2, Concorde Complex, Above OBC Bank. R.C. Dutt Road., Alkapuri Baroda.
Page # 14
Real Numbers (Class X) Mathematics

4. Find the largest number that divides 2053 and 967 and leaves a remainder of 5 and 7 respectively.

5. A mason has to fit a bathroom with square marble tiles of the largest possible size. The size of the
bathroom is 10 ft. by 8 ft. What would be the size in inches of the tile required that has to be cut and
how many such tiles are required?

6. Two brands of chocolates are available in packs of 24 and 15 respectively. If I need to buy an equal
number of chocolates of both kinds, what is the least number of boxes of each kind I would need to
buy?

7. Three sets of English, Hindi and Mathematics books have to be stacked in such a way that all the
books are stored topic-wise and the height of each stack is the same. The number of English books
is 96, the number of Hindi books is 240 and the number of Mathematics books is 336. Assuming
that the books are of the same thickness, determine the number of stacks of English, Hindi and
Mathematics books.

8. A merchant has 120 litres of oil of one kind, 180 litres of another kind and 240 litres of third kind.
He wants to sell the oil by filling the three kinds of oil in tins of equal capacity. What should be the
greatest capacity of such a tin?

9. Find the [HCF × LCM] for the numbers 105 and 120.

10. The product of two numbers is 396 × 576 and their LCM is 6336. Find their HCF.

11. The HCF of two numbers is 119 and their LCM is 11781. If one of the numbers is 1071, find the
other.

12. The LCM of two numbers is 2079 and their HCF is 27. If one of the numbers is 189, find the other.

13. Find the greatest measure which is exactly contained in 10 liters 857 millilitres and 15 litres 87
millilitres.
14. Find the LCM and HCF of 18, 24, 60, 150

15. Find the HCF and LCM of 60, 32, 45, 80, 36, 120

16. What is the greatest number by which 1037 and 1159 can both be divided exactly ?

17. Find the greatest number which both 2458090 and 867090 will contain an exact number of times.

18. Find the number neaerst to 11000 but greatest than 10000 which is exactly divisible by each of 8,
15 and 21

19. Find the largest number that divides 1251, 9377and 15628 leaving remainders 1, 2 and 3, respectively.

20. On a morning walk, three persons step off together and their steps measure 40 cm, 42 cm and 45
cm, respectively. What is the minimum distance each should walk so that each should walk, so
that each can cover the same distance in complete steps?
21. Find the greatest number of a 6 digit exactly divisible by 24, 15 and 36.

ANSWERS
1. 435 2. 204 3. 3647 4. 64 5. 24 Inch, 20 tiles

6. 5,8 7. 2,5,7 8. 60 litres 9. 12600 10. 36

11. 1309 12. 297 13. 141 millilitres 14. 1800, 6

15. 1, 1440 16. 61 17. 10 18. 109200

19. 625 20. 2520 21. 999720

IIT ASHRAM UG–1 & 2, Concorde Complex, Above OBC Bank. R.C. Dutt Road., Alkapuri Baroda.
Page # 15
Real Numbers (Class X) Mathematics

DPP - 5
(QUESTIONS ON TERMINATING AND NON TERMINATING DECIMAL EXPANSION)
1. Without actually performing the long division, state whether the following rational numbers have a
terminating decimal expansion or a non-terminating repeating decimal expansion :

1 1 22 3 7 2 27
(i) (ii) (iii) (iv) (v) (vi) (vii)
7 11 7 5 20 13 40

13 23 42
(viii) (ix) (x)
125 7 100

2. Write down the decimal expansions of the following rational numbers :

241 19 25 9 133
(i) (ii) (iii) (iv) (v)
2352 256 1600 30 2354

3. Without actually performing the long division, state whether the following rational numbers will
have a terminating decimal expansion or a non-terminating repeating decimal expansion :

11 19 32 15 29
(i) (ii) (iii) (iv) (v)
125 128 405 3200 2401

4. Write down the decimal expansions of the following rational numbers :

5 12 13 7 7
(i) (ii) (iii) (iv) (v)
8 125 625 64 8

5. Without actually performing the long division, find if 987/10500 will have terminating or non-
terminating (repeating) decimal expansion. Give reasons for your answer.

6. A rational number in its decimal expansion is 327.7081. What can you say about the prime factors
of q, when this number is expressed in the form p/q ? Give reasons.

7. Without actually performing the long division, state whether the following rational numbers will
have terminating decimal expansion or a non-terminating repeating decimal expansion:
29 15 13 23
(i) (ii) (iii) (iv)
343 1600 3125 2352

ANSWERS
1. (i) Non-terminating repeating (ii) Non-terminating repeating ;
(iii) Non-terminating repeating (iv) Terminating (v) Terminating

(vi) Non-terminating repeating (vii) Terminating (viii) Terminating

(ix) Non-terminating repeating (x) Terminating


2. (i) 1.205 (ii) 0.07421875 (iii) 0.015625 (iv) 0.3 (v) 0.0266

3. (i) Terminating (ii) Terminating


(iii) Non-terminating repeating (iv) Terminating

(v) Non-terminating repeating

4. (i) 0.625 (ii) 0.096 (iii) 0.0208 (iv) 0.109375 (v) 0.875

5. 0.094 7. (i) non-terminating, (ii) terminating, (iii) terminating, (iv) terminating

IIT ASHRAM UG–1 & 2, Concorde Complex, Above OBC Bank. R.C. Dutt Road., Alkapuri Baroda.
Page # 16
Real Numbers (Class X) Mathematics

DPP - 6
(QUESTIONS ON IRRATIONALITY
1. Prove that 5 is an irrational number.
2. Show that the following numbers are irrational.
1
(i) (ii) 7 5 (iii) 6 + 2 (iv) 3 – 5
2
3. Prove that 5  3 is irrational.

4. Prove that 7 is an irrational number.

5. Prove that 3  5 2 is irrational.

6. Show that 5  3 in irrational.

7. Prove that 2  5 is irrational.

8. Prove that 2 is an irrational number.

9. Show that 2  3 in irrational.

10. Prove that 3


3 is an irrational.

7
11. Prove that 2 is not a rational number.
5

12. Prove that 4  5 3 is an irrational.

13. Prove that 3  5 is an irrational number.

1
14. Prove that is an irrational number.
2 3

15. Prove that 3 is an irrational number.

16. Prove that 5  2 3 is an irrational number.

17. Prove that 3  2 is an irrational number.

2 45  3 20
18. Write whether on simplification gives a rational or an irrational number.
2 5

19. Prove that for any prime positive integer p, p is an irrational number.+

20. Prove that a positive integer n is prime, if no prime p less than or equal to n divides n.

IIT ASHRAM UG–1 & 2, Concorde Complex, Above OBC Bank. R.C. Dutt Road., Alkapuri Baroda.
Page # 17
Real Numbers (Class X) Mathematics

DPP - 7
(QUESTIONS ON SURDS)

(1) If 10  12  a  2 b , then find the values of a and b. (a, b  Q)

(2) If 20  6 5  a  2 b , then find the values of a and b. (a, b  Q)

(3) If 7  5  a  2 b , then find the values of a and b. (a, b  Q)

(4) State the conjugate surd of 9  7

(5) State the conjugate surd of 5 – 3

(6) Express 4  20 in the form of a  2 b

(7) Express 17  6 2 in the form of a  2 b

(8) Express 4  7 in the form of a  2 b

(9) If 3  b  a  2 5 , then find the values of a and b. (a, b  Q)

(10) Find the square root of 15  2 54

(11) Find the square root of 7  48

(12) Find the square root of 3  2 2

(13) Find the square root of 17  2 72

(14) Evaluate: 7  2 12  7  2 12

(15) Evaluate: 52 6  52 6

ANSWERS

5
1. a = 10, b = 3 2. a = 20, b = 45 3. a  7,b 
4

7
4. 9 7 5. 5 3 6. 42 5 7. 17  2 18 8. 42
4

9. a = 3, b = 20 10. 3  6 11. 2  3 12. 2 1

13. 3  8 14. 4 15. 2 2

IIT ASHRAM UG–1 & 2, Concorde Complex, Above OBC Bank. R.C. Dutt Road., Alkapuri Baroda.
Page # 18
Real Numbers (Class X) Mathematics

Find the square root of the following:

(1) 8  2 15 (2) 7  40 (3) 30  224 (4) 13 – 120

(5) 15  6 6 (6) 19  8 3 (7) 10  84 (8) 6  35

(9) 3 5

ANSWERS

1. 5 3 2. 5 2 3. 2 7– 2 4. 10  3 5. 3 6 6. 4 3

7 5 5 1
7. 7 3 8.  9. 
2 2 2 2

Solve the following:

(1) Find the fourth root of 28  16 3 (2) Find the fourth root of 49  20 6

(3) Find the fourth root of 56  12 20 (4) Find the fourth root of 17  12 2

7 3
(5) Find the fourth root of 7  4 3 (6) Find the fourth root of  5
2 2

(7) Prove that: 94 5  94 5 4 (8) Prove that: 8  2 15  8  2 15  2 5

1 1
(9) Prove that:  2 2
5 2 6 5 2 6

ANSWERS

3 1
1. 3 1 2. 3 2 3. 5 1 4. 2 1 5. 
2 2

5 1
6. 
2 2

IIT ASHRAM UG–1 & 2, Concorde Complex, Above OBC Bank. R.C. Dutt Road., Alkapuri Baroda.
Page # 19
Real Numbers (Class X) Mathematics

MCQ EXERCISE - 1
1. If a is any positive integer and p is a prime 10. Euclid’s division lemma states that for two
number such that p divides a2 then p will also positive integers a and b, there exist unique
divide integers q and r such that a = bq + r. where
1
(a) (b) a r must satisfy:
a 3

(c) a (d) None of these. (a) 0  r < b (b) 1 < r < b

2. For some integer ‘m’ every even integer is of (c) 0 < r  b (d) 0 < r < b
the form 11. The decimal expansion of the rational number
(a) 2m – 1 (b) 2m + 1 61
will terminate after:
24  54
(c) 2(m + 1) (d) m

3. For some integer ‘m’, every odd integer is of (a) 1 place (b) 2 places

the form: (c) 3 places (d) 4 places

(a) m (b) 2m + 1 (c) m + 1 (d) 3m +1 12. If HCF of 210 and 55 is expressed in the form

4. The largest number which divides 70 and 125 of 210 × 5 + 55y, then value of y2 is

leaving remainder 5 and 8 respectively is (a) 381 (b) 368 (c) 361 (d) 19

(a) 11 (b) 13 (c) 17 (d) None 13. The least number that is divisible by all the
numbers from 1 to 10 (both inclusive) is
131
5. The decimal form of 3. 2. 1 is: (a) 90 (b) 100 (c) 2520 (d) 360
2 5 7

(a) terminating 14 For any positive integer n, n3 – n is always


divisible by
(b) non-terminating non-repeating
(a) 7 (b) 6
(c) non-terminating repeating
(c) 5 (d) None of these
(d) none of three
15. The product of a non-zero rational number and
6. Number 6n can not end with
irrational number is
(a) 6 (b) 5
(a) always irrational
(c) Can not say (d) None of these.
(b) always rational
7. n2 – 1 is divisible by 8 if n is:
(c) one
(a) an integer (b) an even integer
(d) rational or irrational
(c) a natural number (d) an odd integer
16. If the HCF of 85 and 153 is expressible in the
8. Which of the following is not an irrational form 85 m – 153, then value of m is:
number:
(a) 1 (b) 2 (c) 3 (d) 4
(a) 3  3 (b) 4  27
17. The decimal expansion of the rational number
(c) 9  27 (d) 46  46 97
will terminate after;
2  54
9. A rational number has non-terminating
repeating decimal; if prime factors of its (a) One decimal place
denominator are not of the form. (b) Two decimal place
m m m m
(a) 2 × 5 (b) 3 × 5 (c) Three decimal place
m m
(c) 2 × 3 (d) None of these. (d) Four decimal place

IIT ASHRAM UG–1 & 2, Concorde Complex, Above OBC Bank. R.C. Dutt Road., Alkapuri Baroda.
Page # 20
Real Numbers (Class X) Mathematics

18. If x and y are odd positive integers then x2 + y2 (c) natural number
is not divisible by (d) an integer
(a) 1 (b) 2 20.  is a/an
(c) 4 (d) None of these (a) Natural number

19. 6  3 3    4  5 3  is a (b) irrational number


(c) Not defined
(a) rational number

(b) irrational number 22


(d)
7

ANSWER KEY
1. (c) 2. (c) 3. (b) 4. (b) 5. (c)
6. (b) 7. (d) 8. (d) 9. (a) 10. (a)
11. (d) 12. (c) 13. (c) 14. (b) 15. (a)
16. (b) 17. (d) 18. (c) 19. (b) 20. (b)

IIT ASHRAM UG–1 & 2, Concorde Complex, Above OBC Bank. R.C. Dutt Road., Alkapuri Baroda.
Page # 21
Real Numbers (Class X) Mathematics

MCQ EXERCISE - 2
11. (5k + 1)2 leaves remainder ....... on dividing by
1. Product of any four consecutive positive
integers is divisible by ......... . 5.

(a) 16 (b) 48 (c) 24 (d) 32 (a) 2 (b) 0

2. If g.c.d of two numbers is 8 and their product is (c) -1 or +1 (d) 1

384 then their L.C.M. is ....... 12. On division by 6, a2 cannot leave remainder

(a) 24 (b) 16 (c) 48 (d) 32 ........ a  N

3. If p1 and p2 are distinct primers, their g.c.d. (a) 1 (b) 4 (c) 5 (d) 3

is....... . 13. Product of three consecutive integers is


divisible by .......
(a) p1 (b) P2 (c) p1p2 (d) 1
(a) 24 (b) 8 but not by 24
4. 3  5 = .......
(c) 6 (d) 20
(a) 3 2 (b) 5 1
14587
14. has ....... digits after decimal point.
1250
5 1
(c) (d) does not exist
2 (a) 5 (b) 4 (c) 3 (d) 2

5. If g.c.d. (a, b) = 1 then g.c.d. 15. Which is terminating decimal number ?

(a - b, a + b) = ........ 125 77
(a) (b)
441 210
(a) 1 or 2 (b) a or b

(c) a + b or a - b (d) 4 15 129


(c) (d)
1600 25  72  5 2
6. If n > 1, n4 + 4 is a ........, n  N
16. Which number is terminating decimal ?
(a) a prime

(b) a composite number 37 21 17 89


(a) (b) (c) (d)
45 2356 41 22  32
(c) 1 (d) infinite

7. If g.c.d. (a, b) = 18, L.C.M. (a, b)  ...... 17. (2  5 ) is ....... number

(a) 36 (b) 72 (c) 48 (d) 108 (a) rational (b) irrational

2517 (c) non real solution (d) integer


8. The decimal expansion of will terminate
6250
18. ....... is conjugate surd of 8 7
after ........ digits.

(a) 4 (b) 5 (c) 3 (d) 6 (a) 7  8 (b) 7 8

9. 2m 5n (m  N, n  N) ends with (c) 7  8 (d) 7  8


(a) 0 (b) 5 (c) 25 (d) 125
19. 5  2 6  ..........
317
10. represents ........ (a) 3 2 (b) 3  2
3125

(a) a terminating decimal (c) 3 2 (d) 3  2

(b) non recurring decimal 20. 9  44  a  2 b then a = ....... , b = .......


(c) a recurring decimal (a) 3, 11 (b) 9, 11 (c) 9, 44 (d) 3, 22
(d) an integer

IIT ASHRAM UG–1 & 2, Concorde Complex, Above OBC Bank. R.C. Dutt Road., Alkapuri Baroda.
Page # 22
Real Numbers (Class X) Mathematics

21. 8  2 15 = ....... 2009


31. convert into decimal represen-tation.
24  56
(a) 5 3 (b) 5 3
(a) 0.08036 (b) 0.008036
(c) 5 3 (d) 5  3
(c) 0.0008036 (d) None
22. g.c.d. (65,117) = 65m– 117 then m = ......
32. 7  1 = .........
(a) 4 (b) 2 (c) 1 (d) 3
(a) 7 1 (b) 7 1
23. g.c.d. (96, 404) = 4 then L.C.M. = ........

(a) 9696 (b) 9096 (c) 9600 (d) 9008 14  2 14  2


(c) (d)
24. Number ....... is the least number which is 2 2
divided from the numbers 1 to 10. 33. L.C.M. (610, 89) = ........
(a) 10 (b) 100 (c) 504 (d) 2520 (a) 610 (b) 89 (c) 1 (d) 54290
25. Product of two consecutive numbers is always 34. g.c.d. (52, 320) = ........
divisible by ......... (a) 8 (b) 4 (c) 5 (d) 1
(a) 2 (b) 3 (c) 4 (d) 5 35. g.c.d. (280, 674) = ........
26. Which number is irrational ? (a) 2 (b) 4 (c) 14 (d) 8
(a)  2 (b)  3
36. The sum of the exponents of the prime factors
(c) (a) and (b) both (d) None in the prime factorisation of 196 is ........
27. p and q are natural numbers such that p = qm (a) 1 (b) 2 (c) 4 (d) 6
+ n then ......... 37. m = 23 × 3, d = 2 × 3 × 5, p = 3n × 5 L.C.M. (m, d,
(a) 0  n  q (b) 0  n  q p) = 23 × 32 × 5 then n = .........
(c) 0  p  n (d) n > q (a) 1 (b) 2 (c) 3 (d) 4
28. L.C.M (x, y, z) = ........ 38. Rationalising factor of 27 is ........

xyz (a) 27 (b) 3 3 (c) 3 (d) 3


(a) g.c.d.(x, y)  g.c.d.(y, z)  g.c.d(x, z)

39. 7  40 = .........
xyz  g.c.d.(x, y)
(b) g.c.d.(x, y, z)  g.c.d(y, z) (a) (b)
5 2 5 2

xyz  g.c.d.(y, z) (c) 3 2 (d) 3 2


(c) g.c.d.(x, y, z)  g.c.d(x, y)  g.c.d.(x, z)
40. The L.C.M. and g.c.d. of two rational numbers
are equal then numbers must be .........
xyz
(d) (a) prime (b) co-prime
g.c.d.(xy, yz, zx)

29. g.c.d of (32, 66, 89) = ........ (c) composite (d) equal

(a) 4 (b) 3 (c) 2 (d) 1 41. The smallest positive number divisible by 24,36
and 48 is ………..
30. 6  2 5 = ........ (a) 48 (b) 96 (c) 144 (d) 288
(a) 2 1 (b) 3  5 42. g.c.d. (28, 35, 91)= ………….

(c) 5 1 (d) 5 1
(a) 1 (b) 5 (c) 7 (d) 14

IIT ASHRAM UG–1 & 2, Concorde Complex, Above OBC Bank. R.C. Dutt Road., Alkapuri Baroda.
Page # 23
Real Numbers (Class X) Mathematics

43. The smallest positive number divisible by every 48. If LCM (a, b) = x, GCD (a, b) = y then ab =
integer from 2 to 6 is …………
(a) x/y (b) x + y
(a) 12 (b) 24 (c) 30 (d) 60
(c) xy (d) x – y
44. The numbers in the form 3k  1 k  Z  are
49. 9  141  ………….
(a) even numbers (b) odd numbers
(a) does not exist (b) 9  141
(c) divisible by 3 (d) not divisible by3
(c) –( 9  141 ) (d) 141  9
45. Every even integer a is in the form of ………….;
50. The product of prime factors of 180 is …
where k  Z.
(a) 2 × 2 × 5 × 9 (b) 4× 9 × 5
(a) 2k (b) 2k + 1
(c) 2 × 2 × 3 × 3 × 5 (d) 10 × 18
(c) 5k (d) 3k + 1
51. If n is positive even integer, then n (n + 1)
46. Every odd in te ge r a i s in the form of
(n + 2) is
………where k  Z.
(a) a prime number (b) divisible by 20
(a) k + 1 (b) 2k + 1
(c) divisible by 24 (d) divisible by 16
(c) 3k (d) 4k + 2
1
52. The decimal expansion of is ______
47. 17  12 2 = ……… 32
(a) 0.03125 (b) 0.15625
(a) 3 2 (b) 3  2 2
(c) 0.3125 (d) 0.00625
5 1
(c) (d) does not exist
2

ANSWER KEY
1. c 2. c 3. d 4. c 5. a
6. b 7. c 8. b 9. a 10. a
11. d 12. c 13. c 14. b 15. c
16. b 17. b 18. d 19. a 20. b
21. b 22. b 23. a 24. d 25. a
26. c 27. a 28. d 29. d 30. c
31. b 32. c 33. d 34. b 35. a
36. c 37. b 38. c 39. a 40. d
41. c 42. c 43. d 44. d 45. a
46. b 47. b 48. c 49. a 50. c
51. c 52. a

IIT ASHRAM UG–1 & 2, Concorde Complex, Above OBC Bank. R.C. Dutt Road., Alkapuri Baroda.
Page # 24
Real Numbers (Class X) Mathematics

CLASSROOM ASSIGNMENT

ASSIGNMENT - 1
1. Use Euclid’s division algorithm to find the HCF of :
(i) 135 and 225 (ii) 196 and 38220 (iii) 867 and 255
2. Show that any positive odd integer is of the form 6q + 1, or 6q + 3, or 6q + 5, where q is some integer.
3. An army contingent of 616 members is to march behind an army band of 32 members in a parade.
The two groups are to march in the same number of columns. What is the maximum number of
columns in which they can march?
4. Use Euclid’s division lemma to show that the square of any positive integer is either of the form 3m
or 3m + 1 for some integer m.
[Hint : Let x be any positive integer then it is of the form 3q, 3q + 1 or 3q + 2. Now square each of
these and show that they can be rewritten in the form 3m or 3m + 1.]
5. Use Euclid’s division lemma to show that the cube of any positive integer is of the form 9m, 9m + 1
or 9m + 8.

ANSWERS
1. (i) 45 (ii) 196 (iii) 51
2. An integer can be of the form 6q, 6q + 1, 6q + 2, 6q + 3, 6q + 4 or 6q + 5.
3. 8 columns
4. An integer can be of the form 3q, 3q + 1, 3q + 2, Square all of these integers.
5. An integer can be of the form 9q, 9q + 1, 9q + 2, 9q + 3,...... or 9q + 8.

ASSIGNMENT - 2
1. Express each number as a product of its prime factors:
(i) 140 (ii) 156 (iii) 3825 (iv) 5005 (v) 7429
2. Find the LCM and HCF of the following pairs of integers and verify that LCM × HCF = product of the
two numbers.
(i) 26 and 91 (ii) 510 and 92 (iii) 336 and 54
3. Find the LCM and HCF of the following integers by applying the prime factorisation method.
(i) 12, 15 and 21 (ii) 17, 23 and 29 (iii) 8, 9 and 25
4. Given that HCF (306, 657) = 9, find LCM (306, 657).
5. Check whether 6n can end with the digit 0 for any natural number n.
6. Explain why 7 × 11 × 13 + 13 and 7 × 6 × 5 × 4 × 3 × 2 × 1 + 5 are composite numbers.
7. There is a circular path around a sports field. Sonia takes 18 minutes to drive one round of the field,
while Ravi takes 12 minutes for the same. Suppose they both start at the same point and at the
same time, and go in the same direction. After how many minutes will they meet again at the
starting point?

IIT ASHRAM UG–1 & 2, Concorde Complex, Above OBC Bank. R.C. Dutt Road., Alkapuri Baroda.
Page # 25
Real Numbers (Class X) Mathematics

ANSWERS
1. (i) 22  5  7 (ii) 22  3 13 (iii) 32  52 17 (iv) 5  7 1113 (v) 17 19  23
2. (i) LCM = 182; HCF = 13 (ii) LCM = 23460; HCF = 2
(iii) LCM = 3024; HCF = 6
3. (i) LCM = 420; HCF = 3 (ii) LCM = 11339; HCF = 1
(iii) LCM = 1800; HCF = 1
4. 22338 7. 36 minutes

ASSIGNMENT - 3
1. Prove that 5 is irrational.
2. Prove that 3  2 5 is irrational

3. Prove that the following are irrationals:

1
(i) (ii) 7 5 (iii) 6 2
2

ASSIGNMENT - 4
1. Without actually performing the long division, state whether the following rational numbers will
have a terminating decimal expansion or a non-terminating repeating decimal expansion

13 17 64 15 23 129
(i) (ii) (iii) (iv) (v) 3 2 (vii) 2 7 5
3125 8 455 1600 2 5 2 5 7

6 35 77
(viii) (ix) (x)
15 50 210

2. Write down the decimal expansions of those rational numbers in Question 1 above which have
terminating decimal expansions.
3. The following real numbers have decimal expansions as given below. In each case, decide whether
they are rational or not. If they are rational, and of the form , p
q what can you say about the prime factors of q?
(i) 43.123456789 (ii) 0.120120012000120000. . .
(iii) 43.123456789

ANSWERS
1. (i) Terminating (ii) Terminating (iii) Non-terminating repeating
(iv) Terminating (v) Non-terminating repeating (vi) Terminating
(vii) Non-terminating repeating (viii) Terminating (ix) Terminating
(x) Non-terminating repeating
2. (i) 0.00416 (ii) 2.125 (iv) 0.009375 (vi) 0.115 (viii) 0.4
(ix) 0.7
3. (i) Rational, prime factors of q will be either 2 or 5 or both only.
(ii) Not rational
(iii) Rational, prime factors fo q will also have a factor other than 2 or 5.

IIT ASHRAM UG–1 & 2, Concorde Complex, Above OBC Bank. R.C. Dutt Road., Alkapuri Baroda.
Page # 26
Real Numbers (Class X) Mathematics

HOTS

1. Simplify: 12  2 27  12  2 27
1 1
2. 
Simplify: 49  2 600   49  2
4 600 4

2 2 4
3. Simplify:
 
8  2 15 42 3 62 5
1 1 2
4. Simplify:
 
12  140 8  60 10  84

5. Solve the following:

1
(i) If x  9  4 5 , then find the value of x
x

(2) Simplify: 2  3  4 3  2 8 3 8

3
(3) If 1  a  b 3 , then prove that a + b = 0. (a, b  Q)
4

a a  2b
6. Let a and b be positive integers. Show that 2 always lies between and .
b ab
1 2

7. Let a, b, c, p be rational numbers such that p is not a perfect cube. If a  b p 3  cp 3  0 , then


Prove that a = b = c = 0.
8. If d is the HCF of 56 and 72, find x, y satisfying d = 56x + 72y. Also, show that x and y are not
unique.

9. Show that there is no positive integer n for which n  1  n  1 is rational.

10. Let a,b, c, d be positive rationals such that a  b  c  d then either a = c and b = d or b and
d are squares of rationals

ANSWERS
1. 0 2. 2 3 3. 0 4. 0
5. (i) 1. 4 (ii) 2

IIT ASHRAM UG–1 & 2, Concorde Complex, Above OBC Bank. R.C. Dutt Road., Alkapuri Baroda.
Page # 27
Real Numbers (Class X) Mathematics

OLYMPIAD
OLYMPIAD DRILL - 1
10. The sum of three non-zero prime numbers is
1. The least perfect square number which is 100. One of them exceeds the other by 36. Find
divisible by 8, 15,20,22 is– the largest number.
(a) 435600 (b) 43560 (a) 73 (b) 91 (c) 67 (d) 57
(c) 39600 (d) None 11. If N is the sum of first 13,986 prime numbers,
2. Th e g reatest n umber wh ich can divi de then N is always divisible by–
1854,1866 an d 2066 le avin g th e same (a) 6 (b) 4 (c) 8 (d) None
remainder 2 in each case is–
12. The least number which is a perfect square
(a) 4 (b) 6 (c) 12 (d) None and is divisible by each of 16, 20 and 24 is
3. The greatest number of five digits which on (a) 240 (b) 1600 (c) 2400 (d) 3600
being divided by 56,72, 84 and 96 leaves 50,
13. Find the least number which when divided by
66, 78 and 90 as remainders is–
12, leaves a remainder of 7, when divided by
(a) 98784 (b) 98778 (c) 98790 (d) None 15, leaves a remainder of 10 and when divided
4. When 2 256
is divided by 17 the remainder would by 16, leaves a remainder of 11
be– (a) 115 (b) 235 (c) 247 (d) 475
(a) 1 (b) 16 (c) 14 (d) None 14. If n is an even natural number, then the
5. The sum of three non-zero prime numbers is largest natural number by which n (n + 1)(n +
100. One of them exceeds the other by 36. Find 2) is divisible is
the largest number. (a) 6 (b) 8 (c) 12 (d) 24
(a) 73 (b) 91 (c) 67 (d) 57 15. Find the least number which when divided by
6. If N is the sum of first 13,986 prime numbers, 15, leaves a remainder of 5, when divided by
then N is always divisible by 25, leaves a remainder of 15 and when divided
by 35 leaves a remainder of 25
(a) 6 (b) 4 (c) 8 (d) None
(a) 515 (b) 525 (c) 1040 (d) 1050
7. H.C.F. of (x3 – 3x + 2) and (x2 – 4x + 3) is–
16. If (–1 )n + (–1 )4n = 0, then n is–
(a) (x – 1) (b) (x – 2)2
(a) any positive integer
(c) (x – l)(x + 2) (d) (x – l)(x – 3)
(b) any negative integer
8. If two numbers when divided by a certain
divisor give remainder 35 and 30 respectively (c) any odd natural number
and when their sum is divided by the same (d) any even natural number
divisor, the remainder is 20, then the divisior
17. The number 313 – 310 is divisible by
is
(a) 2 and 3 (b) 3 and 10
(a) 40 (b) 45 (c) 50 (d) 55
(c) 2, 3 and 10 (d) 2, 3 and 13
9. In order that the number 1 y 3 y 6 be divisible
by 11, the digit y should be

(a) l (b) 2 (c) 5 (d) 6

IIT ASHRAM UG–1 & 2, Concorde Complex, Above OBC Bank. R.C. Dutt Road., Alkapuri Baroda.
Page # 28
Real Numbers (Class X) Mathematics

18. A number lies between 300 and 400. If the 19. What is the number x
number is added to the number formed by
I. The LCM of x and l8 is 36.
reversing the digits, the sum is 888 and if the
unit’s digit and the ten’s digit change places, II. The HCF of x and 18 is 2.
the new number exceeds the original number (a) l (b) 2 (c) 3 (d) 4
by 9. Find the number.
(a) 339 (b) 341 (c) 378 (d) 345

ANSWER KEY
1. a 2. a 3. b 4. a 5. c 6. d
7. a 8. b 9. c 10. c 11. d 12. d
13. b 14. d 15. a 16. c 17. d 18. d
19. c

OLYMPIAD DRILL - 2 4. The value of

1. The descending order of the surds 3


2, 6 3, 9 4 1 1 1
is ________ .   
1 2 2 3 3 4
9 1 1 1 1 1
(a) 4 , 6 3, 3 2 (b) 9
4 , 3 2, 6 3    
4 5 5 6 6 7 7 8 8 9
(c) 3
2 , 6 3, 9 4 (d) 6
3 , 9 4, 3 2
is____.
2. The smallest between
(a) 0 (b) 1 (c) 2 (d) 4
17  12 and 11  6 is ________
5. If a  b  c  d , where a, b, c, d are
(a) 17  12 consecutive natural numbers, then whcih of
the following is true?
(b) 11  6
(a) a b  c  d
(c) both are equal
(b) c d a b
(d) Can’t be determined
(c) a c b d
3. A circular field has a circumference of 360 km.
(d) c d a b
Two cyclists Sumeet and Samrat start together
and can cycle at speed of 12 km/hr and 15km/ 1
hr respectively round the circular field. After 6. If t  , then t is equal to ____ .
1 4 2
how many hours will they meet again at the
starting point? (a) 1  2  2  2 
4
(b) 1  2  2  2 
4

(a) 60 hrs (b) 8 hours (c) – 1  2 1  2 


4
(d) 1  2  2  2 
4

(c) 100 hrs (d) 120 hrs


The 100th root of1010  is____
10
7.

(a) 10810 (b) 10108

10
 10  10
(c)  10  (d) 10 10  

IIT ASHRAM UG–1 & 2, Concorde Complex, Above OBC Bank. R.C. Dutt Road., Alkapuri Baroda.
Page # 29
Real Numbers (Class X) Mathematics

a  a 2 – b2 a  a 2  b2 13. x1, x2, x3, ...., x10 are integers, none of which
8. The value of  is__
a  a 2  b2 a  a 2 – b2
are divisible by 3. The remainder when
x12  x 22  x 23  .......  x10
2
is divided by 3 is.

(a)
a2
(b)
b2
(c)
a
(d)

2 2a 2  b2  (a) 0 (b) 0 or 2
b2 a2 b b2
(c) 1 or 2 (d) 1
9. The rationalising factor of 2 3 5 is _____.
14. An eight digit number is a multiple of 73 and
(a) 3 (b) (c) 52 (d) 53
5 3
52 137. If the second digit from left is 7, then 6th
10. If a679b is a five-digit number in base 10 and digit from the left of the number is ____.
is divisible by 72, then the value of a and b (a) 7 (b) 9 (c) 5 (d) 3
are_________
15. According to the Fundamental Theorem of
(a) 3, 2 (b) 6, 2 (c) 2, 3 (d) 2, 6 Arithmetic, if p(a prime number) divides b2, and

11. The rationalising factor of b is positive, then ______.


7
x 3 y 5 z 2 is_____.
(a) b divides p (b) b2 dividees p
(a) 7 5
z y x2 4
(b) 4 3
x y z 2
(c) p2 divides b2 (d) p divides b
(c) x 4 y2z5 (d) 3
y 2 x4 z3

2 6
12. is equal to
2 3 5

(a) 2 3 5 (b) 4  2  3

1
(c) 2  3  6 5 (d)
2
 2 5– 3 

ANSWER KEY
1. c 2. b 3. d 4. c
5. b 6. c 7. b 8. d
9. b 10. a 11. a 12. a
13. d 14. a 15. d

IIT ASHRAM UG–1 & 2, Concorde Complex, Above OBC Bank. R.C. Dutt Road., Alkapuri Baroda.
Page # 30
Real Numbers (Class X) Mathematics

OLYMPIAD DRILL - 3 8. If x, y, z be rational numbers such that x > y


and z < y then
1. If ‘m’ is an irrational number then ‘2m’ is__
(a) z > x (b) z < x (c) y < z (d) y > x
(a) rational number
9. For any two rational numbers x and y, which of
(b) an irrational number
the following properties are correct?
(c) a whole number
(i) x<y (ii) x = y (iii)x > y
(d) a natural number
(a) only (i) and (ii) are correct
2. The sum of a rational and an irrational number
(b) Only (ii) and (iii) are correct
is____
(c) Only (ii) is correct
(a) an irrational number
(d) All
(b) a rational number
10. If A : The quotient of two integers is always a
(c) an integer
1
(d) a whole number rational number and R : is not rational
0
3. The product of two irrationals is then which of the following statements is true?

(a) a rational number (a) A is true and R is the correct explanation

(b) an irrationl number of A

(c) either A or B (b) A is False and R is the correct explanation


of A
(d) neither A nor B
(c) A is true and R is false
4. The value of 1.34  4.12 is
(d) Both A and R are false
133 371 5169 5411
(a) (b) (c) (d) 11. HCF(p, q, r) . LCM (p, q, r) =
99 90 990 990

5. The value of pq qr
(a) (b)
r p
5
4
1 (c) p, q, r (d) None of these
1
1
3
1 12. If 3
32  2 x then x is equal to
2
4
3 5
40 4 1 31 (a) 5 (b) 3 (c) (d)
(a) (b) (c) (d) 5 3
31 9 8 40
13. The greatest possible number which when
6. The greater among the following is divided by 37 and 58, leaves the respective
–2 remainder of 2 and 3, is -
3 1 1 17
I. 3
1.728 II. III.   IV.
3 1 2 8 (a) 2 (b) 5

(a) I (b) IV (c) II (d) III (c) 10 (d) None of these

7. The two irrational numbers between 2 and 14. The largest possible number with which when
3 are 60 and 98 are divided, leaves the remainder 3
in each case, is
1 1 1 1 1 1
(a) 2 ,6 2 4 (b) 3 ,34 6 (c) 6 ,3 (d) None
8 4
(a) 38 (b) 18

(c) 19 (d) None of these

IIT ASHRAM UG–1 & 2, Concorde Complex, Above OBC Bank. R.C. Dutt Road., Alkapuri Baroda.
Page # 31
Real Numbers (Class X) Mathematics

25. If a | b, then gcd of a and b is


15. The largest possible number with which when
38, 66 and 80 are divided the remainders (a) a (b) b
remain the same is – (c) ab
(a) 14 (b) 7 (c) 28 (d) None (d) Can’t be determined
16. What is the least possible number which when 26. If gcd of b and c is g and d | b & d | c, then –
divided by 24, 32 or 42 in each case it leaves
(a) d = g (b) g | d
the remainder 5 ?
(c) d | g (d) None of these
(a) 557 (b) 677 (c) 777 (d) None
27. If x and y are positive real numbers, then
17. In Q.N. 16, how many numbers are possible
between 666 and 8888 ? (a) x y  xy

(a) 10 (b) 11 (c) 12 (d) 13 (b) x y  xy

18. What is the least number which when divided (c) x y  xy
by 8, 12 and 16 leaves 3 as the remainder in (d) None of these
each case, but when divided by 7 leaves no
15
remainder ? 28. is equal to
10  20  40  125
(a) 147 (b) 145 (c) 197 (d) None

19. What is the least possible number which when (a) 5 (5  2 ) (b) 5 (2  2 )
divided by 18, 35 or 42 leaves 2, 19, 26 as the
(c) 5 ( 2  1) (d) 5 (3  2 )
remainders respectively ?

(a) 514 (b) 614 (c) 314 (d) None 29. 2  3  2  3 is equal to
20. What is the least possible number which when
divided by 2, 3, 4, 5, 6 leaves the remainders 3 2
(a) 3 (b) (c) (d) 6
1, 2, 3, 4, 5 respectively ? 2 3
(a) 39 (b) 48 (c) 59 (d) None
3 1
21. In Q.No. 20, what is the least possible 3 digit 30. The expression is equal to
number which is divisible by 11 ? 2 2  3 1

(a) 293 (b) 539 (c) 613 (d) None (a) 2 3 4 6


22. How many numbers lie between 11 and 1111 (b) 6 4 3 2
which when divided by 9 leave a remainder of
(c) 6 4 3 2
6 and when divided by 21 leave a remainder of
12 ? (d) None of these

(a) 18 (b) 28 (c) 8 (d) None 31. The total number of divisors of 10500 except 1
and itself is –
23. If x divides y (written as x | y) and y | z, (x, y, z
 z) then (a) 48 (b) 50 (c) 46 (d) 56

(a) x | z (b) z | y 32. The sum is the factors of 19600 is

(c) z | x (d) None (a) 54777 (b) 33667

24. If x | y, where x > 0, y > 0 (x, y  z) then (c) 5428 (d) None of these

(a) x < y (b) x = y (c) x  y (d) x  y

IIT ASHRAM UG–1 & 2, Concorde Complex, Above OBC Bank. R.C. Dutt Road., Alkapuri Baroda.
Page # 32
Real Numbers (Class X) Mathematics

33. The product of divisors of 7056 is –

(a) (84)48 (b) (84)44

(c) (84)45 (d) None of these


34. The number of odd factors (or divisors) of 24 is

(a) 2 (b) 3 (c) 1 (d) None


35. The number of even factors (or divisors) of 24
is

(a) 6 (b) 4 (c) 8 (d) None


36. In how many ways can 576 be expressed as a
product of two distinct factors ?

(a) 10 (b) 11 (c) 21 (d) None

37. 910 blue pens and 1001 red pens are distibuted
to students of class X so that each student gets
the same number of pens of each kind. What
is the maximum strength of the class ?

(a) 91 (b) 80 (c) 94 (d) 86

38. A positive number ‘n’ when divided by 8 leaves


a remainder 5. What is the remainder when
2n + 4 is divided by 8?

(a) 8 (b) 1 (c) 6 (d) 0

ANSWER KEY
1. b 2. a 3. c 4. d 5. c 6. d
7. c 8. b 9. d 10. b 11. d 12. d
13. b 14. c 15. a 16. b 17. d 18. a
19. b 20. c 21. b 22. a 23. a 24. c
25. a 26. c 27. a 28. d 29. b 30. a
31. c 32. a 33. c 34. a 35. a 36. a
37. a 38. c

IIT ASHRAM UG–1 & 2, Concorde Complex, Above OBC Bank. R.C. Dutt Road., Alkapuri Baroda.
Page # 33

Вам также может понравиться